Đến nội dung

BoY LAnH LuNg nội dung

Có 79 mục bởi BoY LAnH LuNg (Tìm giới hạn từ 16-05-2020)



Sắp theo                Sắp xếp  

#566687 Tìm m để phường trình $x^{2}-2(m+2)+m^{2}-4=0$...

Đã gửi bởi BoY LAnH LuNg on 18-06-2015 - 17:07 trong Đại số

Cho phương trình $x^{2}-2(m+2)+m^{2}-4=0$. Tìm các giá trị của m để phường trình đã cho co 2 nghiệm phân biệt $x_{1},x_{2}$ thỏa mãn $\left | x_{1} \right |-4\left | x_{2} \right |=0$

 

Chú ý:  Cách gõ công thức Toán.

             Cách đặt tiêu đề bài viết đúng quy định.

giải $\Delta$

áp dụng hệ thức Vi-ét thay x1, x2 vào




#566686 Cho đường thẳng (d) có phương trình 2(1-m)x+(2-m)y+2=0 (m là tham số)

Đã gửi bởi BoY LAnH LuNg on 18-06-2015 - 17:05 trong Đại số

$a)$ $(d)$ đi qua $A(2;1)$ --> $2(1-m).2+(2-m).1+2=0$ -> giải $m$

$b)$ Giả sử họ đường thẳng $d)$ đi qua 1 điểm cố định $M(a;b)$ thì $2(1-m)a+(2-m)b+2=0$ với mọi $m$ --> $-m(2a+b)+(2a+2b+2)=0$ với mọi $m$

--> $\left\{\begin{matrix}2a+b=0 \\ 2a+2b+2=0 \end{matrix}\right.\Leftrightarrow \left\{\begin{matrix}a=1 \\ b=-2 \end{matrix}\right.$

$c)$ $\rightarrow$ cắt trục tung tại $(0;$\frac{2}{m-2}$)$ và cắt trục hoành tại $($\frac{1}{m-1}$;0)$ --> khoảng cách $(d)$ tới $O$ là $d'$ --> $\frac{1}{d'^{2}}=(m-1)^{2}+\frac{(m-2)^{2}}{4}$--> giải $Min$  

giải cụ thể đi bạn




#566684 Tìm giá trị của m để $y_{1}^{2}+y_{2}^...

Đã gửi bởi BoY LAnH LuNg on 18-06-2015 - 17:03 trong Đại số

Trong mặt phẳng tọa độ Oxy, cho parabol (P): $y=x^{2}$ và đường thẳng (d): $y=mx+2$, với m là tham số. 

1. CMR: Với mọi giá trị của tham số m, đường thẳng (d) luôn cắt parabol (P) tại 2 điểm phân biệt.

2. Gọi $A(x_{1};y_{1})$ và $B(x_{2};y_{2})$ là giao điểm (d) và (P). Tìm giá trị của m để $y_{1}^{2}+y_{2}^{2}$ đạt GTNN.

1. dùng phương trình hoành độ giao điểm

2. thay x vào. áp dụng vi-et với phương trình hoành độ giao điểm để thay x bằng m




#566676 1.Cho 2015 số nguyên dương phân biệt không vượt quá 3019. Chứng minh trong 20...

Đã gửi bởi BoY LAnH LuNg on 18-06-2015 - 16:56 trong Số học

 

2. Cho $x,y,z$ là các số thực dương, nhỏ hơn 1 thỏa mãn $xyz=(1-x)(1-y)(1-z)$. Chứng minh trong ba số $x(1-y),y(1-z),z(1-x)$ có một số không nhỏ hơn $\frac{1}{4}$

giả sử cả 3 số đều nhỏ hơn 0,25

VT < 1/64

VP > 9/64   -----> vô lý




#566674 $T=2+2 \sqrt{12n^{2}+1)$

Đã gửi bởi BoY LAnH LuNg on 18-06-2015 - 16:52 trong Số học

Chứng minh rằng nếu $T=2+2 \sqrt{12n^{2}+1)$ là số tự nhiên thì $T$ là số chính phương

http://diendantoanho...-t22-sqrt12n21/

có rùi mờ




#566672 Cho dãy số $1,3,2,1,-1,2,3,a,b,....$

Đã gửi bởi BoY LAnH LuNg on 18-06-2015 - 16:50 trong Số học

Cho dãy số $1,3,2,1,-1,2,3,a,b,....$

Tìm $a,b$

hiện

1+2=3

3-1=2

2+(-1)=1.....

a=-1

b=-4




#566661 Tìm hai số nguyên dương $x,y (x>y>0)$ thỏa mãn hai số...

Đã gửi bởi BoY LAnH LuNg on 18-06-2015 - 16:37 trong Số học

$x^{2}< x^{2}+3y< x^{2}+2x+1+x-1$

vậy $x^{2}+3y=(x+1)^{2}$

???????




#566653 $y^2-4x-3=0 $

Đã gửi bởi BoY LAnH LuNg on 18-06-2015 - 16:28 trong Số học

vậy bài này..!!

y^2=36-2x (18>=x)

x,y nguyên k




#566649 Choose the correct answer and explain your choice

Đã gửi bởi BoY LAnH LuNg on 18-06-2015 - 16:24 trong CLB Ngoại ngữ (English, Francais, Ruskʲə)

I have two sentences below . Choose the correct answer and explain your choice

1) Even if you are rich , you should save some money for a ........... day

a. foggy       b.  snowy        c. windy         d. rainy

2) Tim had a blazing ........ with Sue and stormed  out of the house . 

a. chat         b. gossip         c. row            d. word

1. c

2. d




#498360 Chứng minh:$\sum \frac{x^{3}}{y+2z...

Đã gửi bởi BoY LAnH LuNg on 11-05-2014 - 10:34 trong Bất đẳng thức và cực trị

k gõ LATEX ai mà hiểu được

MEM chú ý nên báo ĐHV để sửa tiêu đề không trả lời topic vi phạm




#497934 Tìm số tự nhiên n nhỏ nhất (n>4) sao cho n3+4n2-20n-48 chia hết cho 125

Đã gửi bởi BoY LAnH LuNg on 08-05-2014 - 22:31 trong Số học

Tìm số tự nhiên n nhỏ nhất (n>4) sao cho n3+4n2-20n-48 chia hết cho 125




#495003 $\boxed{\text{Chuyên Đề}}$ Bất đẳng thức - Cực trị

Đã gửi bởi BoY LAnH LuNg on 24-04-2014 - 22:50 trong Bất đẳng thức và cực trị

156, a, Cho x,y thỏa $4x^{2}+3y^{2}=\frac{3}{28}.Tìm GTLN của M=\left | 2x+5y \right |$

b, Cho x, y > 0 thoả $\frac{2}{x}+\frac{3}{y}=6.Tìm GTNN của P=x+y$

c, Cho $x\epsilon \left [ 0;1 \right ].Tìm GTLN của P=x(1-x)^{3}$

d, Cho $x\epsilon \left [ 0;2 \right ] và y\epsilon \left [ 0;\frac{1}{2} \right ]. Tìm GTLN của Q=(2x-x^{2})(y-2y^{2})$

e, Cho $x\epsilon \left [ 0;3 \right ] và y\epsilon \left [ 0;4 \right ].Tìm GTLN của S=(2-x)(3-2y)(2x+3y)$

 

Chú ý: Không kẹp $$ vào tiếng Việt có dấu




#495000 $\boxed{\text{Chuyên Đề}}$ Bất đẳng thức - Cực trị

Đã gửi bởi BoY LAnH LuNg on 24-04-2014 - 22:41 trong Bất đẳng thức và cực trị

155, cho a,b,c là 3 cạnh một tam giác. CMR

a, $\sum \frac{a^{2}+2bc}{b^{2}+c^{2}}> 3$

b, nếu $a\leq b\leq c  thì  (a+b+c)^{2}\leq 9bc$




#492696 Cho $a,b,c,d\in \mathbb{Z}$ và $a,b,c,d...

Đã gửi bởi BoY LAnH LuNg on 13-04-2014 - 17:48 trong Số học

đã có http://diendantoanho...4d2014là-hợp-s/




#492462 Tìm giá trị nhỏ nhất của P = $\frac{x^{2}}...

Đã gửi bởi BoY LAnH LuNg on 12-04-2014 - 17:31 trong Bất đẳng thức và cực trị

Cho các số thực a, b, c >0 thoả mãn ab + bc + ca = 1

Tìm Min của P = $\frac{x^{2}}{x + y} + \frac{y^{2}}{y + z} + \frac{z^{2}}{z + x}$

sao đề cho a,b,c mà yêu cầu là x, y, z




#492461 $\boxed{\text{Chuyên Đề}}$ Bất đẳng thức - Cực trị

Đã gửi bởi BoY LAnH LuNg on 12-04-2014 - 17:25 trong Bất đẳng thức và cực trị

 $cho a,b.c dương và abc = 1 cmr a + b + c \leq ab +bc +ca$

nhớ đánh số thứ tự




#492175 Topic tổng hợp các bài toán về phương trình nghiệm nguyên.

Đã gửi bởi BoY LAnH LuNg on 11-04-2014 - 17:40 trong Số học

218, giải các phương trình nghiệm nguyên

a, $8x+11y=73$

b, $5x-3y=2xy-11$

c, $x^{2}+(x+1)^{2}+(x+2)^{2}=y^{2}$

d, $x^{3}+y^{3}+z^{3}=2003^{4}$

e, xyz = 9 + x + y + z và x, y, z >0

g, $x^{3}-x^{2}-2xy=y^{3}+y^{2}+100$

h, 5 (x + y + z + t) + 10 = 2xyzt  và x, y, z, t là các số dương

i, 2 + 3x = 5x với x không âm

k, 19x2 + 28y2 = 729 với x, y nguyên dương

l, 9x + 5 = y(y+1)

m, 2016x + 3 = y3

n, 2x2 + 4x = 19 - 3y2

p, x4 + 2x3 + 2x2 + x + 3 = y2

r, x3 + 2y3 = 4z3 (sử dụng phương pháp lùi vô hạn)

s, x3 + y3 + z3 = (x + y + z)2 với x, y, z đôi một khác nhau

t, x3 - y3 = xy + 8

u, 6x + 15y + 10z = 3

v, 2x + 57 = y2

w, 12x+ 6xy + 3y2 = 28(x + y)




#491943 Topic về các bài toán lớp 6

Đã gửi bởi BoY LAnH LuNg on 10-04-2014 - 17:18 trong Đại số

Kết quả hình như là $\frac{17}{25}$ phải ko bn




#491570 Topic về các bài toán lớp 6

Đã gửi bởi BoY LAnH LuNg on 09-04-2014 - 09:02 trong Đại số

 

Bài 37: Tính

a) $A=\frac{8}{9}.\frac{15}{16}.\frac{24}{25}...\frac{2499}{2500}$

 

ta có thể phân tích $A=\frac{(2.4)(3.5)(4.6)...(49.51)}{(3.3)(4.4)(5.5)...(50.50)}=\frac{(2.3.4.5...49)(4.5.6...51)}{(3.4.5...50)(3.4.5...50)}$

đến đây giản ước là xong




#491426 Tìm giá trị nhỏ nhất của biểu thức $P=(1+\frac{a}{b...

Đã gửi bởi BoY LAnH LuNg on 08-04-2014 - 17:52 trong Bất đẳng thức và cực trị

Sao phải rắc rối thế hả!!

Áp dụng BĐT $AM-GM$ ta được:

$(1+\frac{a}{b})(1+\frac{b}{c})(1+\frac{c}{a})\geq 8\sqrt{\frac{abc}{abc}}=8$

a, b, c không bằng nhau mà




#491425 $\boxed{\text{Chuyên Đề}}$ Bất đẳng thức - Cực trị

Đã gửi bởi BoY LAnH LuNg on 08-04-2014 - 17:49 trong Bất đẳng thức và cực trị

167, cho x,y,z là các số dương thoả x+y+z=xy+yz+zx

CM  $\frac{1}{x^{2}+y+1}+\frac{1}{y^{2}+z+1}+\frac{1}{z^{2}+x+1}$ $\leq 1$

áp dụng bdt Bunyakovsky ta có

$(x^{2}+y+1)(1+y+z^{2})\geq (x+y+z)^{2}\Rightarrow \frac{1}{x^{2}+y+1}\leq \frac{1+y+z^{2}}{(x+y+z)^{2}}$

tương tự $\frac{1}{y^{2}+z+1}\leq \frac{1+z+x^{2}}{(x+y+z)^{2}}$

$\frac{1}{z^{2}+x+1}\leq \frac{1+x+y^{2}}{(x+y+z)^{2}}$

cộng 3 bất dẳng thức trên ta được 

$\frac{1}{x^{2}+y+1}+\frac{1}{y^{2}+z+1}+\frac{1}{z^{2}+x+1}\leq \frac{3+x+y+z+x^{2}+y^{2}+z^{2}}{(x+y+z)^{2}}$

ta chỉ cần cm 

$3+x+y+z+x^{2}+y^{2}+z^{2}\leq (x+y+z)^{2}$

$\Leftrightarrow 3+xy+yz+zx+x^{2}+y^{2}+z^{2}\leq (x+y+z)^{2}$

$\Leftrightarrow 3\leq xy+yz+zx$

do $x+y+z=xy+yz+zx$ và $xy+yz+zx\leq \frac{(x+y+z)^{2}}{3}$

nên $xy+yz+zx\leq \frac{(xy+yz+zx)^{2}}{3}$

$\Rightarrow xy+yz+zx\geq 3$

bài toán dc cm

 

 

p/s bài này mình làm rồi hi hi  :icon6:  :lol:  :wacko:




#491417 $\boxed{\text{Chuyên Đề}}$ Bất đẳng thức - Cực trị

Đã gửi bởi BoY LAnH LuNg on 08-04-2014 - 17:14 trong Bất đẳng thức và cực trị

Bài 165:

Cho các số thực $a,b,c$ thuộc khoảng $(0;1)$ thoả mãn $abc=(1-a)(1-b)(1-c)$.

Chứng minh rằng: $a^2+b^2+c^2\geq \frac{3}{4}$

 

P/s: Mình thấy bài này cũng khá hay, hình như giống đề thi thử đại học năm nay của tỉnh Vĩnh Phúc thì phải. Mọi người làm thử xem sao nhé!

$abc=(1-a)(1-b)(1-c)\Rightarrow abc=1-(a+b+c)+ab+bc+ca-abc$

$\Rightarrow 2abc=1-(a+b+c)+ab+bc+ca(1)$

Lại có $abc=(1-a)(1-b)(1-c)$ và $a,b,c\epsilon (0,1)$

$\Rightarrow 1=(\frac{1}{a}-1)(\frac{1}{b}-1)(\frac{1}{c}-1)$

áp dụng bdt côsi

$(\frac{1}{a}-1)+(\frac{1}{b}-1)+(\frac{1}{c}-1)\geq 3\sqrt[3]{(\frac{1}{a}-1)(\frac{1}{b}-1)(\frac{1}{c}-1)}$

$(\frac{1}{a}-1)+(\frac{1}{b}-1)+(\frac{1}{c}-1)\geq 3$

$\Rightarrow \frac{1}{a}+\frac{1}{b}+\frac{1}{c}\geq 6$

$\Rightarrow ab+bc+ca\geq 6abc$(2)

$(1)(2)\Rightarrow ab+bc+ca\geq 3-3(a+b+c)+3(ab+bc+ca)$

$\Rightarrow 0\geq 3-(a+b+c)+2(ab+bc+ca)$

$\Rightarrow a^{2}+b^{2}+c^{2}\geq (a+b+c)^{2}-3(a+b+c)+3$

$\Rightarrow x^{2}+y^{2}+z^{2}\geq (x+y+z-\frac{3}{2})^{2}+\frac{3}{4}\geq \frac{3}{4}$

suy ra dpcm




#491281 $\boxed{\text{Chuyên Đề}}$ Bất đẳng thức - Cực trị

Đã gửi bởi BoY LAnH LuNg on 07-04-2014 - 18:50 trong Bất đẳng thức và cực trị

164, Cho $a,b,c\geq 0$ thoả mãn $a^{2}+b^{2}+c^{2}=1$

CMR $\frac{2a+c}{1+bc}+\frac{2b+c}{1+ca}+\frac{a+b+c}{1+\sqrt{2}abc}\leq 3\sqrt{2}$

ta cần cm $\frac{2a+c}{1+bc}+\frac{2b+c}{1+ca}\leq 2\sqrt{2}$  và  $\frac{a+b+c}{1+\sqrt{2}abc}\leq \sqrt{2}$

ta có * $\frac{2a+c}{1+bc}\leq \frac{8a+4c}{2\sqrt{2}(a+b+c)}$

TT $\frac{2b+c}{1+ca}\leq \frac{8b+4c}{2\sqrt{2}(a+b+c)}$

cộng theo vế có đpcm

* $\frac{a+b+c}{1+\sqrt{2}abc}\leq 2 \Leftrightarrow a+b+c-2abc\leq \sqrt{2}$

 

$a+b+c-2abc=a(1-2bc)+(b+c)$

áp dụng bdt bunhiacopxki ta có $a+b+c-2abc=a(1-2bc)+(b+c)\leq \sqrt{[a^{2}+(b+c)^{2}].[(1-2bc)^{2}+1]}=\sqrt{(1+2ab)(2-4ab+4a^{2}b^{2})}$

do đó ta cần cm $\sqrt{(1+2ab)(2-4ab+4a^{2}b^{2})}\leq 2\Leftrightarrow 8a^{3}b^{3}-4a^{2}b^{2}\leq 0\Leftrightarrow ab\leq \frac{1}{2}$

BDT này luôn đúng do $ab\leq \frac{a^{2}+b^{2}}{2}\leq \frac{a^{2}+b^{2}+c^{2}}{2}=\frac{1}{2}$

suy ra dpcm




#491270 $\boxed{\text{Chuyên Đề}}$ Bất đẳng thức - Cực trị

Đã gửi bởi BoY LAnH LuNg on 07-04-2014 - 18:19 trong Bất đẳng thức và cực trị

162, cho 3 số thực a, b, c thoả mãn $a+b+c\leq 3$

Tìm GTLN của $P=\sum \frac{a+1+a .\sqrt{a^{2}+1}}{\sqrt{a^{2}+1}}$

$P=\frac{a+1}{\sqrt{a^{2}+1}}+\frac{b+1}{\sqrt{b^{2}+1}}+\frac{c+1}{\sqrt{c^{2}+1}}+a+b+c$

Mà $\frac{a+1}{\sqrt{a^{2}+1}}\leq \sqrt{2}$

$\frac{b+1}{\sqrt{b^{2}+1}}\leq \sqrt{2}$

$\frac{c+1}{\sqrt{c^{2}+1}}\leq \sqrt{2}$

$a+b+c\leq 3$

nên $P\leq 3\sqrt{2}+3$

Dấu = xẩy ra khi $a=b=c=1$




#491266 $\boxed{\text{Chuyên Đề}}$ Bất đẳng thức - Cực trị

Đã gửi bởi BoY LAnH LuNg on 07-04-2014 - 18:00 trong Bất đẳng thức và cực trị

161, Cm BDT $\sqrt{a^{2}+b^{2}}+\sqrt{c^{2}+d^{2}}\geq \sqrt{(a+c)^{2}+(b+d)^{2}}$